0
$\begingroup$

Given a sequence of signed measures $<\nu_j>$, if it happens that $\nu=\sum\limits_{j = 1}^\infty \nu_j$ is still a valid signed measure (then it can be proved that each partial sum $\nu_n=\sum\limits_{j = 1}^n \nu_j$ is valid signed measure), do we have $\lim\limits_{n\to \infty}|\sum\limits_{j = 1}^n \nu_j|=|\sum\limits_{j = 1}^\infty \nu_j|$ ($=|\lim\limits_{n\to \infty} \sum\limits_{j = 1}^n \nu_j|$)? Thanks!

$\endgroup$
3
  • 3
    $\begingroup$ Which topology do employ on the space of measures? The one induced by total variation norm? $\endgroup$
    – shuhalo
    Feb 19, 2011 at 8:46
  • $\begingroup$ And measures on which space, by the way ? $\endgroup$
    – BS.
    Feb 19, 2011 at 12:19
  • $\begingroup$ Sorry for the late response. I'm concerned with the equality in the post which looks like a continuity property, so I used the term "continuous" in the title. Of course it would be better if we can establish a setting in which topology is equipped and then continuity of total variation of signed measures has the usual meaning. But currently I have no idea about this. $\endgroup$
    – zzzhhh
    Feb 21, 2011 at 10:59

2 Answers 2

1
$\begingroup$

Assuming I understood the question correctly, the answer is no. Consider measures on $\{0,1\}^\omega$ with the product topology and Borel $\sigma$-algebra. Let $\mu_i$ be the uniform measure on the set with $i$-th coordinate equal to 0. This sequence converges by your definition to the uniform measure, but all $\mu_i$ are far (in total variation) from the uniform measure. (To fit your description we can take $\nu_i=\mu_i-\mu_{i-1}$).

$\endgroup$
4
  • $\begingroup$ Sorry but what is the uniform measure? $\endgroup$
    – zzzhhh
    Feb 21, 2011 at 11:00
  • 1
    $\begingroup$ The uniform measure is the probability measure of total measure 1, and for each $i$, $\mathbb{P}(x_i=0)=\frac12$ independently. $\endgroup$ Feb 21, 2011 at 18:57
  • $\begingroup$ I refrained from asking what is total measure. Could you please refer me to a textbook containing conceptions like uniform measure, total measure so that I can understand your reply after reading it? Thanks! $\endgroup$
    – zzzhhh
    Feb 22, 2011 at 10:40
  • 1
    $\begingroup$ I'm really nor very good with books recommendations, but any rigorous book on probability theory should be helpful. Perhaps the following reformulation will help: the space is just $I=[0,1]$. $\mu$ is the Lebesgue measure restricted to $I$. $I_n$ is the set of all reals in $I$ whose $n$-th binary digit is 0. $\mu_n$ is twice the Lebesgue restricted to $I_n$, i.e. $\mu_n(A)=2\mu(A \cap I_n)$. Then for every measurable $A$ we have $\mu_n(A) \to \mu(A)$, but the total variation distance between any $\mu_n$ and $\mu$ is 1. $\endgroup$ Feb 22, 2011 at 18:22
1
$\begingroup$

Am I misunderstanding your question? Because it seems to me that if you consider the $\nu_n = \frac{(-1)^n}{n} \delta$ where $\delta$ is the Dirac mass, then we've got the usual situation with the alternating harmonic series.

$\endgroup$
1
  • $\begingroup$ Yes, this is an example that the equality holds. But do we have counterexamples? $\endgroup$
    – zzzhhh
    Feb 21, 2011 at 11:00

Your Answer

By clicking “Post Your Answer”, you agree to our terms of service and acknowledge you have read our privacy policy.

Not the answer you're looking for? Browse other questions tagged or ask your own question.